Find the coordinates of the midpoint of a segment with the endpoints (9.–3) and (5,1).

Answers

Answer 1

Answer:

(7,-1)

Step-by-step explanation:

Answer 2

Answer:

(7,-1)

Step-by-step explanation:

Midpoint = [(x1+x2)÷2] , [(y1+y2)÷2]

= [(9+5)÷2] , [(-3+1)÷2]

= (7 ,-1)


Related Questions

What is the midpoint of AB, with endpoints A(3,2) and B (8,14)

Answers

Answer:

Step-by-step explanation:

(3 + 8)/2= 11/2

(2 + 14)/2= 16/2= 8

(11/2, 8) the midpoint

A rectangular room is 11 feet by 18 feet. The ceilings are 10 feet high. If a gallon of paint covers 250 square feet. How many gallons of paint are needed to paint the room?

Answers

Answer:

2.32 gallons of paint

Step-by-step explanation:

First, find the area of all 4 sides of the room, with the equation A = lw

11(10)

= 110 (for 2 of the walls)

18(10)

= 180 (for the other 2 walls)

Add these up:

110 + 110 + 180 + 180

= 580

Then, divide this by 250:

580/250

= 2.32

So, 2.32 gallons of paint will be needed to paint the room.

Petra is shopping with 2 of her friends. She buys a note book and six identical pencils. The note book costs the same as 2 of the pencils. Her bill is £16.80. How much does a note book cost?

Answers

Answer: £4.20

Each 2 of pencils is £4.20 multiply that by 3 your get 12.60, the notebook is the same price of 2 pencils so that must mean it is £4.20

4.20 x 4 = 16.80

3 £4.2 from the 6 pencils

and 1 £4.2 from the notebook

Hope this helps!

The required cost of a notebook would be an amount of £4.20.

What is the equation?

The equation is defined as mathematical assertions that have a minimum of two terms containing variables or numbers that are equal.

Let's assume that n is the cost of a notebook, p is the cost of a pencil, and b is the total bill.

We know that the notebook costs the same as two pencils, so we can write this as an equation:

n = 2 × p

We also know that Petra's bill is £16.80, so we can write this as an equation:

b = n + 6 × p

Substituting the first equation into the second equation, we get:

b = n + 6 × p

b = (2 × p) + 6 × p

b = 8 × p

We can now solve for p by dividing both sides of the equation by 8:

p = b / 8

p = £16.80 / 8

p = £2.10

Since the notebook costs the same as two pencils, we can now solve for n by substituting the value of p into the first equation:

n = 2 × p

n = 2 × £2.10

n = £4.20

Therefore, the cost of a notebook is £4.20.

To learn more about the equations click here:

brainly.com/question/10413253

#SPJ3

Find the distance between points P(5, 1) and 2(3, 4) to the nearest tenth.

Answers

Answer:

using distance formula =√(5-3)sq+(1-4)sq

=√4+9

=√13

Step-by-step explanation:

-3(4a + 3) + 4(6x + 1) = 43

Answers

Answer:

-3(4a+3)+4(6x+1)

12a+(-9)+24x+5

12a-4+24x

Step-by-step explanation:

Because you can't add 12a & 24x bc they're different variables ( a & x ) & -9+5 you always minus it and it's going to be a negative because -9 is bigger than 5

i need a step by step explanation of how to solve this

Answers

Answer: x = 6

Step-by-step explanation:

Subtract 8 on both sides. The new equation should be -2x = -3x + 6.Now, add 3x on both sides. It should give you x = 6.

Gina has 2/3 pounds of apples. Jane has 3/5 pound of apples. Gina and Jane want to combine their apples to make an apple pie. The recipe calls for 1 1/2 pounds of apples. How many pounds of apples do they need for their pie? Show all work.

Answers

Answer:

Step-by-step explanation:

1. 3x5=15

2. 2x5=10 and 3x3=9

3 10/15 + 9/15= 19/15 or 1 and .3 so they need approx .2 more pounds

Identify the pattern for the following sequence. Find the next three terms in the sequence.
-1, 3,-9, 27,
a. Multiply by 3; 81, 243, 729
b. Multiply by -3; -81, 243, -729
C. Multiply by -3; 81, 243, 729
d. Multiply by 3; -81, -243, -729

Answers

The answer is B.

-9/3= -3, which means that each number is getting multiples by -3. Therefore, we can eliminate A and D. 27•-3=-81, so we can eliminate C. We are left with B, and we can check this by making sure that each number is correctly multiplied by -3.

Answer:

answer is b

Step-by-step explanation:

53, -82, 103, -7, 22, 0 in ascending order

Answers

Answer:

-82, -7, 0, 22, 53, 103

Step-by-step explanation:

smaller to bigger

In the bottom question what does X= to?

Answers

Answer:

The value of x is 29°...

When an angle is bisected, it is cut in half. We can assume that m<BAR = m<CAB and we know that m<CAR = m<CAB + m<BAR

88 = 2x - 14 + 2x - 14

Combine like terms

88 = 4x - 28

Add 28 to both sides

116 = 4x

Divide both sides by 4

29 = x

The picture gives information about a car traveling at a constant speed.
Which choice shows the car's speed between checkpoints A and B, expressed as a
unit rate?
Checkpoint A
Checkpoint B
Time 100 pm
Time: 2:30 pm
Total Mies Driver 120
Total Miles Daven 195

Answers

Answer:

Step-by-step explanation:

Answer

Answer:

50 miles per hour is the (Unit Rate)

Step-by-step explanation:

ASAP
Water flowed out of a tank at steady rate. A total of 18 1/2 gallons flowed out of a tank in 4 1/4 hours. Which expression determines the the quality of water leaving the tank per hour?
O 17/4 divide 36/2
O 36/2 divide 17/4
O 17/4 divide 37/2
O 37/2 divide 17/4

Answers

Answer: In 1 hour water flowed out = 74/17  gallons

Step-by-step explanation:

Given as ,

The quantity of water flowed out of tank =  18 1/2 gallons

                                                                  =   37/2   gallons

The time spend in flowing water out of tank = 4  1/4 hours

                                                                         =   17/4   hours

∵ In   17/4 hours , water flowed out = 37/2

∴ In    1    hours                           ,  water flowed out =

So, in 1 hour water flowed out =  37*4/17*2 gallons Answer

Answer:

total galloons flowed=18 1/2=36/2

time that it flowed=4 1/4=16/4

rate=total galloons flowed/time to flow

rate=36/2÷16/4

=9/2gallons per hour

4.5gallons per hour

factorise 4a + 8b - 24

Answers

Answer: 4(a+2b−6)

Hope this helps!

A basketball player averages 12.5 points per game. There are 24 games in a season. At this rate, how many points would the player score in an entire season?

Answers

Answer:

300

Step-by-step explanation:

hope this helps

Can someone please help me?​

Answers

Answer:

no

Step-by-step explanation:

u r a n00b hahahahahahahahahahahahahaahahhaahahahahahahahahahahahahahahahahahahhahahau suk balz hhahahahahahahahahahahahahahaha

I NEED HELP 11POINTS
Casey has a small business making and selling dessert baskets. She estimates that her
fixed weekly costs to bake the items is $100. She sells each dessert basket for $12.
How much would her monthly profit be for x amount of dessert baskets?

Answers

Answer:

500=2.5x+200

x=the amount of desserts.

500-200=2.5x

300=2.5x

x=120

Step-by-step explanation:

Which Angel number supplementary to angel to AED

Answers

Answer:

Angle number 5

Step-by-step explanation:

The sum of supplementary angles equal 180°.

Angle AED, in the diagram given is labelled angle 4.

The angle labelled 4 forms a linear pair with the angle labelled 5.

Since a linear pair of angles = 180°, therefore:

m<AED + m<AEI = 180°.

That is:

m<4 + m<5 = 180°.

Both angles are supplementary.

Therefore, angle number 5 is supplementary to angle AED.

what is the answer to 1/4 - 2/5

Answers

Answer:

0.15

Step-by-step explanation:

longerrrrrrrrrrrrr

A point lies on AB¯¯¯¯¯ and is located 3/7 the distance from A to B. What are the coordinates of the point? (4,​ 3) (7,​ 3) (9,​ 3) (12,​ 3)

Answers

Answer:

(4,3)

Step-by-step explanation:

This doesn't even take any math really, just deductive reasoning :)

We know the line segment stretches from one to 15, meaning there are 14 points on the line as there is (1,2,3, etc.)

14/2 obviously equals 7, so just plug it in from there.

12 is too far up on the line to be less than half, so (12, 3) is wrong

^ this goes for (9, 3) as well

7 is literally the halfway marker, so there is no way it could be (7, 3)

Making the answer (4, 3) because not only is it the last one remaining but it makes the most sense, being less than one half of the segment


Write this series in sigma notation: 5+9+13+17+21+ 25 + 29+ 33.

Answers

Answer:

5+9+13+17+21+25+29+33= 152

Answer:

5+9+13+17+21+25+29+33=148

Sarah has $400 in a savings account at the beginning of the summer. She wants to have more than $150 left in her account by the end of the summer. She withdraws $25 per week. How many weeks can she withdraw money from her account and still make her goal? Write an inequality to express the situation

Answers

Answer:

10 weeks; x >= $150

Step-by-step explanation:

400 - 150 = 250

She can only withdraw up to 250.

250 = 25w

250/25 = w

10 = w

10 weeks

Read the paragraph from The Hot Zone.

Just then, he noticed a blur of motion on his left, and he turned and saw two Hazleton workers walking toward him. They weren't supposed to be in here! The area was supposed to be sealed off, but they had come in by another route that led through a storeroom. They wore respirators, but nothing covered their eyes. When they saw the two men in space suits, they froze, speechless. Jerry could not see their mouths, but he could see their eyes, wide with astonishment. It was as if they had suddenly discovered that they were standing on the moon.

What is the author’s main purpose for including this paragraph?

to inform readers about the location of the scene that is unfolding
to explain to readers that being a scientist can be exciting work
to entertain readers by illustrating how dramatic the situation is
to reassure readers that the situation was completely under control

Answers

Answer:

c

Step-by-step explanation:

to entertain readers by illustrating how dramatic the situation is

The purpose of the author will be C. entertain readers by illustrating how dramatic the situation is.

It should be noted that a theme simply means the main idea that's conveyed in a literary work. It's simply what the author wants the readers to know.

In this case, the author’s main purpose for including this paragraph is to entertain readers by illustrating how dramatic the situation is.

Learn more about excerpts on:

https://brainly.com/question/2140963

Write 5.12 x 10⁵ in standard form.

MATH​

Answers

10^5 = 100,000

5.12 × 10^5 in standard form = 5.12 × 100,000 = 512000

Answer: 512,000

Answer:

10^5 = 100,000

5.12 × 10^5 in standard form = 5.12 × 100,000 = 512000

Answer: 512,000

Step-by-step explanation:

Using your equation if the number of square feet is 2400, what is the price of the house? If algebraic work is not shown, describe how you found your answer. How well does this fit with the actual data?

Answers

Answer:

Step-by-step explanation:

Need help. Not understanding

What is the value of x?

Answers

Answer:

98 degrees

Step-by-step explanation:

First, find the value of the other angle of the triangle.

180-53-45 = 82

Then, subtract from 180 to find x. We do this because they both lie on the same line.

180 - 82 = 98

Every line segment has a length ?

Answers

Answer:

A line segment has two endpoints. A line has no end point. A line-segment has a definite length. But a line does not have definite length.

Step-by-step explanation:

f(x)=3x-2 for D={2,3,4}

Answers

are you trying to find x?

What is N equal to? I think I know I want to make sure tho :/

Answers

Answer:

n = 5/7 or 0.7

Step-by-step explanation:

The goal is to isolate the variable N. First, you distribute the fractions to their repsective parentheses.

1/2(5n + 8) = 3/4(12 - 6n)

2.5n + 4 = 9 - 4.5n

2.5n + 4.5n = 9 - 4

7n = 5

Divide both sides by 7, and you get n = 5/7 or 0.7

What is the domain and range of It’s
h(x)= -1/2(x-4)^2 +3

Answers

Answer:

Domain:  [ − 3 ,∞ )

Range:  ( − ∞ , 0 ]

Step-by-step explanation:

h ( x ) = − √ x + 3  

the domain:

x +3 ≥ 0  x ≥ − 3  

in interval form:

[ − 3 , ∞ )  

the range:

lowest possible value under the square root sign is zero, thus:

y ≤ 0  

in the interval form:

( − ∞ , 0 ]

Which has a larger value: 4 x 10^2 or 7 x 10^4?

Answers

Answer:

7 x 10^4

Step-by-step explanation:

4 x 10^2 equals to 400, when 7 x 10^4 equals to 70,000

Answer:

[tex]7[/tex]× [tex]10^{4}[/tex]

Step-by-step explanation:

Because the exponent means that the number is being multiplied by itself that many times. Like [tex]10^{4}[/tex] means 10000 and that times 7 is 70000.

[tex]10^{2}[/tex] would be 100 and 100 times 4 is 400. Now to determine is 400 or 70000 larger? 70000 is larger.

Hope that helps and have a great day!

Other Questions
I need helpppp with this question 5. Why does a properly adjusted head restraint help prevent head and neck injuries to occupants inrear-end collisions? Explain your answer in terms of the law of conservation of momentum. What is 1 x 10^3 in standard form? Show work of how you did it please :) How did the law protect the citizens of the Babylonian Empire? i dont get this question. Please write out the seven elements of music and create a pneumonic to remember them by. Example of a pneumonic: The line notes of the treble clef are EGBDF a pneumonic for them is "Every Good Boy Deserves Fudge" Andrea is selling candles for a fundraiser. She spent $50 on supplies for making the candles. She plans to sell thecandles for $10 each. The function of her profit can be modeled by c(x) = 10x - 50. The x represents the numberof candles she makes. She only has enough supplies to make 100 candles. What is the independent variable? Imagine you were having a discussion with your friend, Alex about the differences between a scientific theory and scientific law. Alex told you he knows that a scientific law is a generalization, or principal that often describes how the natural world behaves under certain conditions, however he was unsure how to describe a scientific theory. Of the choices listed below, which one is least likely accurate in describing a scientific theory? (SC.7.N.3.1) A. Theories include observations. B. Theories have been tested many times. C. Theories are used to make predictions. D. Theories are incomplete, temporary ideas. How are the Celsius, Kelvin, and Fahrenheit scales similar? How are they different? 10 pengamatan lagu indonesia jaya More work was done. Which statement best describes the cause? aThere was more force applied; therefore, there was less movement.bThere was less force applied; therefore, there was more movement.OcThere was less force applied; therefore, there was less movement.OdThere was more force applied; therefore, there was more movement. Is being a multitasker a skill or an ability? what is a nonvoting voter 3. The ratio of an object's mass to itsis called the density of the object. why might finding common ground be important to the futre of the united states What is the mass of a liquid having adensity of 1.40 g/mL and a volume of3090 mL? Complete theirconversation with the correct form of ser or estar. Be sure to choose the correct verbAND to conjugate as needed. Correct spelling, use of accents, and capitalization isrequired. Why does the author introduce the article with a reference to The Strange Case of Dr. Jekyll and Mr. Hyde? can you give me a real-world problem that involves multiplying a fraction and a whole number? Add proper punctuation at the end of the sentence I didnt think I would like it, but now I love it